LSAT and Law School Admissions Forum

Get expert LSAT preparation and law school admissions advice from PowerScore Test Preparation.

 Administrator
PowerScore Staff
  • PowerScore Staff
  • Posts: 8937
  • Joined: Feb 02, 2011
|
#47095
Complete Question Explanation
(The complete setup for this game can be found here: lsat/viewtopic.php?t=11700)

The correct answer choice is (E)

Global Could Be True questions, such as this one, are often best left until the end of the game to complete because answering these questions is often much easier after you have seen several solutions to the game (for example, question #4 in this game contains three answers that can be used to eliminate answers in this question). Thus, unless the game is an Identify the Templates or Possibilities game, skip these questions and return to them at the end of the game. If you do attack this question in its natural position, make sure to use the hypotheticals produced by the first two questions. For example, the solution to question #1 eliminates answer choice (A), and the solution to question #2 eliminates answer choice (B). By using that approach, you will have reduced the amount of work you have to do to determine the correct answer.

Answer choice (A): As mentioned above, the solution to question #1 (answer choice (D), shows that T can be scheduled earlier than L. The work in question #2 also proves the same point.

Answer choice (B): As mentioned above, the work in question #2 shows that L can be scheduled earlier than P.

Answer choice (C): This answer choice can be disproved by the following hypothetical:
D10_game #1_#3_diagram 1.png
As shown above, G can be scheduled earlier than T.

Answer choice (D): This answer choice can be disproved by the following hypothetical:
D10_game #1_#3_diagram 2.png
As shown above, W can be scheduled earlier than T.

Answer choice (E): This is the correct answer choice. G must be scheduled for one of the last three days, and, as discussed in the setup, W must be scheduled for one of the first three days. Because W must always be scheduled earlier than G, this is the correct answer choice. If the logic behind this is unclear, try a hypothetical: there is not enough room for G to be scheduled ahead of W because W must also be scheduled ahead of L, and S must be scheduled in the last three days as well.
You do not have the required permissions to view the files attached to this post.
 Strongam
  • Posts: 10
  • Joined: Oct 14, 2017
|
#41192
Hi I was wondering if I could get some help on #3. The only way I get to the right answer (E), is after making hypotheticals for each answer (except for answer A because it is shown to be false in question 1) showing they could be false and are therefore incorrect. This process takes too much time. Any advice on how to speed it up?
 James Finch
PowerScore Staff
  • PowerScore Staff
  • Posts: 943
  • Joined: Sep 06, 2017
|
#41480
Hi Strongam,

With a question like this, asking for an inference unlikely to be spotted on the setup, the best practice is to first eliminate any answer choices you can from prior information (as you did with (A)), then test the remaining ones in order of likelihood based upon the blocks that you've diagrammed.

In this case, we know that G and S have to take up 2 of the 4-5-6 slots, while W is a leading variable in the W-L block. So without even diagramming it out, we can see that putting G at its earliest possible slot, 4, leaves only 2 slots left for 3 variables, G, W and S. We can't have this in a Balanced Basic Linear game like this one, so (E) ends up being our answer.

In order to get to that answer as the first one to test, you'll have to infer the limited slots available when plugging in the different blocks into different positions on the diagram. This does take some practice to see quickly, but you'll improve as you do more of these Linear games. It also helps to make a bracket showing that, in this game, G and S must go somewhere in slots 4-5-6. We diagram it as [G,S] with the brackets extending directly over all possible slots on the master diagram.

Hope this helps!
 z0egreen
  • Posts: 2
  • Joined: Aug 26, 2020
|
#78439
Hi there, how does Q2 eliminate answer choice B?
User avatar
 Stephanie Oswalt
PowerScore Staff
  • PowerScore Staff
  • Posts: 858
  • Joined: Jan 11, 2016
|
#78440
z0egreen wrote:Hi there, how does Q2 eliminate answer choice B?
Hi Zoe,

You can find the full explanation for Q2 here: https://forum.powerscore.com/lsat/viewtopic.php?t=17258. Please let us know if this helps, or if you would still like further clarification. :)

Thanks!

Get the most out of your LSAT Prep Plus subscription.

Analyze and track your performance with our Testing and Analytics Package.